Cyclic

Algebra Level 4

Positive real a a , b b , c c , and d d are such that a + b + c + d = 1 a + b + c + d = 1 , find the minimum of:

a 3 b + c + b 3 c + d + c 3 d + a + d 3 a + b . \frac{a^3}{b + c} + \frac{b^3}{c + d} + \frac{c^3}{d + a} + \frac{d^3}{a + b} .

This is part of the set My Problems and THRILLER


The answer is 0.125.

This section requires Javascript.
You are seeing this because something didn't load right. We suggest you, (a) try refreshing the page, (b) enabling javascript if it is disabled on your browser and, finally, (c) loading the non-javascript version of this page . We're sorry about the hassle.

2 solutions

Ankit Kumar Jain
Feb 15, 2017

The expression is equivalent to :

a 4 a b + a c + b 4 b c + b d + c 4 c d + c a + d 4 d a + d b \dfrac{a^4}{ab + ac} + \dfrac{b^4}{bc + bd} + \dfrac{c^4}{cd + ca} + \dfrac{d^4}{da + db} , Call it S S


By Titu's Lemma ,

S ( a 2 + b 2 + c 2 + d 2 ) 2 a b + a c + a d + b c + b d + c d + a c + b d S \geq \dfrac{(a^2 + b^2 + c^2 + d^2 )^{2}}{ab + ac + ad + bc + bd + cd + ac + bd} .


Consider the numerator ,

By C - S Inequality , ( a 2 + b 2 + c 2 + d 2 ) ( 1 + 1 + 1 + 1 ) ( a + b + c + d ) 2 = 1 (a^ 2 + b^2 + c^2 + d^2)(1 + 1 + 1 + 1) \geq (a + b + c + d)^{2} = 1

Therefore , ( a 2 + b 2 + c 2 + d 2 ) 2 1 16 . . . . . . . . . . . . . . . . . . . . . . . . . ( 1 ) (a^2 + b^ 2 + c^2 + d^2)^{2} \geq \dfrac{1}{16} ......................... (1)


Now , consider the denominator ,

a b + a c + a d + b c + b d + c d + a c + b d = ( a + d ) ( b + c ) + ( a + b ) ( c + d ) ab + ac + ad + bc + bd + cd + ac + bd = (a + d)(b + c) + (a + b)(c + d)

By AM -GM Inequality on terms ( a + d ) , ( b + c ) (a + d) , (b + c)

( a + d ) ( b + c ) ( ( a + d ) + ( b + c ) 2 ) 2 = 1 4 (a + d)(b + c) \leq (\dfrac{(a + d) + (b + c)}{2})^{2} = \dfrac{1}{4}

Similarly ( a + b ) ( d + c ) ( ( a + b ) + ( d + c ) 2 ) 2 = 1 4 (a + b)(d + c) \leq (\dfrac{(a + b) + (d + c)}{2})^{2} = \dfrac{1}{4} .

Therefore, 1 ( a + d ) ( b + c ) + ( a + b ) ( c + d ) = 1 a b + a c + a d + b c + b d + c d + a c + b d 2 \dfrac{1}{(a + d)(b + c) + (a + b)(c + d)} = \dfrac{1}{ab + ac + ad + bc + bd + cd + ac + bd} \geq 2 . . . . . . . . . . . . . . . . . . . . . ( 2 ) ....................(2)


From ( 1 ) and ( 2 ) (1) \text{and} (2) ,

( a 2 + b 2 + c 2 + d 2 ) 2 a b + a c + a d + b c + b d + c d + a c + b d 1 8 \dfrac{(a^2 + b^2 + c^2 + d^2)^{2}}{ab +ac + ad + bc + bd + cd + ac + bd} \geq \dfrac{1}{8} .

a 3 b + c + b 3 c + d + c 3 d + a + d 3 a + b 1 8 \dfrac{a^3}{b + c} + \dfrac{b^3}{c + d} + \dfrac{c^3}{d + a} + \dfrac{d^3}{a + b} \geq \dfrac{1}{8} .


Equality hold when a = b = c = d = 1 4 a = b = c = d = \dfrac{1}{4}

By Hölder's inequality, ( a 3 b + c + b 3 c + d + c 3 d + a + d 3 a + b ) [ ( b + c ) + ( c + d ) + ( d + a ) + ( a + b ) ] ( 1 + 1 + 1 + 1 ) ( a + b + c + d ) 3 \left(\dfrac{a^3}{b+c}+\dfrac{b^3}{c+d}+\dfrac{c^3}{d+a}+\dfrac{d^3}{a+b}\right)[(b+c)+(c+d)+(d+a)+(a+b)](1+1+1+1)\ge(a+b+c+d)^3

( a 3 b + c + b 3 c + d + c 3 d + a + d 3 a + b ) ( 2 ) ( 4 ) 1 \Longleftrightarrow\left(\dfrac{a^3}{b+c}+\dfrac{b^3}{c+d}+\dfrac{c^3}{d+a}+\dfrac{d^3}{a+b}\right)(2)(4)\ge1

a 3 b + c + b 3 c + d + c 3 d + a + d 3 a + b 1 8 \Longleftrightarrow\dfrac{a^3}{b+c}+\dfrac{b^3}{c+d}+\dfrac{c^3}{d+a}+\dfrac{d^3}{a+b}\ge\dfrac{1}{8}

Equality holds if and only if a = b = c = d = 1 4 a=b=c=d=\dfrac{1}{4}

Can you please help me out with Holder's Inequality?

Ankit Kumar Jain - 4 years, 3 months ago

Log in to reply

You can check out here , it is just a statement about sequences that generalizes the Cauchy-Schwarz inequality to multiple sequences and different exponents.

Dexter Woo Teng Koon - 4 years, 3 months ago

Log in to reply

@Dexter Woo Teng Koon Thanks!

Ankit Kumar Jain - 4 years, 3 months ago

I read about Holder's Inequality somewhere else wherein some other form was given to that of this wiki page. If you could tell me how to add a pic , it would be very helpful for me to share with you the difference in the two forms because I am not that good at LATEX.

Ankit Kumar Jain - 4 years, 3 months ago

Log in to reply

@Ankit Kumar Jain I think you can upload your pic to here and you can share the link with us !

Dexter Woo Teng Koon - 4 years, 3 months ago

Log in to reply

@Dexter Woo Teng Koon Is there any other way ? Because I don't know how to link.

Ankit Kumar Jain - 4 years, 3 months ago

Log in to reply

@Ankit Kumar Jain I think you can just post the url here.

Dexter Woo Teng Koon - 4 years, 3 months ago

For this question, i used ( o 3 + p 3 + q 3 + r 3 ) 1 3 ( s 3 + t 3 + u 3 + v 3 ) 1 3 ( w 3 + x 3 + y 3 + z 3 ) 1 3 o s w + p t x + q u y + r v z (o^3+p^3+q^3+r^3)^{\dfrac{1}{3}}(s^3+t^3+u^3+v^3)^{\dfrac{1}{3}}(w^3+x^3+y^3+z^3)^{\dfrac{1}{3}}\ge osw+ptx+quy+rvz

Dexter Woo Teng Koon - 4 years, 3 months ago

0 pending reports

×

Problem Loading...

Note Loading...

Set Loading...